LSAT and Law School Admissions Forum

Get expert LSAT preparation and law school admissions advice from PowerScore Test Preparation.

User avatar
 Dave Killoran
PowerScore Staff
  • PowerScore Staff
  • Posts: 5850
  • Joined: Mar 25, 2011
|
#72710
This game is also discussed in our Podcast, at the 24:24 mark: LSAT Podcast Episode 37: The November 2019 LSAT Logic Games Section


Complete Question Explanation

(The complete setup for this game can be found here: https://forum.powerscore.com/lsat/viewtopic.php?t=31745)

The correct answer choice is (D).

Answer choice (A): This answer cannot occur because from the last rule O must be added before G.

Answer choice (B): This answer cannot occur because from the last rule K and T must be added after G, and thus the latest G can be added is fourth in Template #1 or third in Template #2.

Answer choice (C): This answer cannot occur because when J is added (Template #1), J would have to be first or sixth.

Answer choice (D): This is the correct answer, and can occur under either Template #1 or Template #2.

Answer choice (E): This answer cannot occur because from the last rule T must be added after O and G, and thus the earliest T can be added is third.

Get the most out of your LSAT Prep Plus subscription.

Analyze and track your performance with our Testing and Analytics Package.